1 Introduction and Main Result

Tsallis [7] generalised in 1988 the standard Bolzmann–Gibbs entropy to a non-extensive quantity \( S_q \) depending on a parameter \( q. \) In the quantum version it is given by

$$\begin{aligned} S_q(\rho )=\frac{1-\mathrm{Tr}~\rho ^q}{q-1}\qquad q\ne 1, \end{aligned}$$

where \( \rho \) is a density matrix. It has the property that \( S_q(\rho )\rightarrow S(\rho ) \) for \( q\rightarrow 1, \) where \( S(\rho )=-\mathrm{Tr}~\rho \log \rho \) is the von Neumann entropy. The Tsallis entropy may be written on a similar form

$$\begin{aligned} S_q(\rho )=-\mathrm{Tr}~\rho \log _q(\rho ), \end{aligned}$$

where the deformed logarithm \( \log _q \) is given by

$$\begin{aligned} \log _q x=\int _1^x t^{q-2}\,dt = \left\{ \begin{array}{ll} \displaystyle \frac{x^{q-1}-1}{q-1}&{}\quad q> 1\\ \log x &{}\quad q=1 \end{array}\right. \end{aligned}$$

for \( x>0. \) The deformed logarithm is also denoted the \( q \)-logarithm. The inverse function \( \exp _q \) is called the \( q \)-exponential and is given by

$$\begin{aligned} \exp _q(x)=\left( x(q-1)+1\right) ^{1/(q-1)}\qquad \text {for}\quad x>\frac{-1}{q-1}\,. \end{aligned}$$

The \( q \)-logarithm and the \( q \)-exponential functions converge, respectively, to the logarithmic and the exponential functions for \( q\rightarrow 1. \)

The aim of this article is to generalise Golden–Thompson’s trace inequality [2, 6] to deformed exponentials. The main result is the following:

Theorem 1.1

Let \( A \) and \( B \) be positive definite matrices.

  1. (i)

    If \( 1\le q<2 \) then

    $$\begin{aligned} \mathrm{Tr}~\exp _q(A+B)\le \mathrm{Tr}~\exp _q(A)^{2-q}\bigl (A(q-1) + \exp _q B\big ). \end{aligned}$$
  2. (ii)

    If \( 2\le q \le 3 \) then

    $$\begin{aligned} \mathrm{Tr}~\exp _q(A+B)\ge \mathrm{Tr}~\exp _q(A)^{2-q}\bigl (A(q-1) + \exp _q B\big ). \end{aligned}$$

Notice that for \( q=1 \) we recover Golden–Thomson’s trace inequality

$$\begin{aligned} \mathrm{Tr}~\exp (A+B)\le \mathrm{Tr}~\exp (A)\exp (B). \end{aligned}$$

This inequality is valid for arbitrary self-adjoint matrices \( A \) and \( B. \) However, it is sufficient to know the inequality for positive definite matrices, since the general form follows by multiplication with positive numbers.

2 Preliminaries

We collect a few well-known results that we are going to use in the proof of the main theorem.

The \( q \)-logarithm is a bijection of the positive half-line onto the open interval \( (-(q-1)^{-1},\infty ), \) and the \( q \)-exponential is consequently a bijection of the interval \( (-(q-1)^{-1},\infty ) \) onto the positive half-line. For \( q>1 \) we may thus safely apply both the \( q\)-logarithm and the \(q \)-exponential to positive definite operators. We also notice that

$$\begin{aligned} \frac{d}{dx}\log _q(x)=x^{q-2}\qquad \text {and}\qquad \frac{d}{dx}\exp _q(x)=\exp _q(x)^{2-q}\,. \end{aligned}$$
(1)

The proof of the following lemma is rather easy and may be found in [4, Lemma 5].

Lemma 2.1

Let \( \varphi :\mathcal D\rightarrow \mathcal A_\text {sa} \) be a map defined in a convex cone \( \mathcal D \) in a Banach space \( X \) with values in the self-adjoint part of a \( C^* \)-algebra \( \mathcal A. \) If \( \varphi \) is Fréchet differentiable, convex and positively homogeneous then

$$\begin{aligned} d\varphi (x)h\le \varphi (h). \end{aligned}$$

for \( x,h\in \mathcal D. \)

Let \( H \) be any \( n\times n \) matrix. The map

$$\begin{aligned} A\rightarrow \mathrm{Tr}~\left( H^* A^p H\right) ^{1/p}, \end{aligned}$$

defined in positive definite \( n\times n \) matrices, is concave for \( 0<p\le 1 \) and convex for \( 1\le p\le 2, \) cf. [1, Theorem 1.1]. By a slight modification of the construction given in Remark 3.2 in the same reference, cf. also [3], we obtain that the mapping

$$\begin{aligned} (A_1,\dots ,A_k)\rightarrow \mathrm{Tr}~\left( H_1^*A_1^pH_1+\cdots +H_k^* A_k H_k\right) ^{1/p}, \end{aligned}$$
(2)

defined in \( k \)-tuples of positive definite \( n\times n \) matrices, is concave for \( 0<p\le 1 \) and convex for \( 1\le p\le 2; \) for arbitrary \( n\times n \) matrices \( H_1,\dots ,H_k. \)

3 Deformed Trace Functions

Theorem 3.1

Let \( H_1,\dots ,H_k \) be matrices with \( H_1^*H_1 +\cdots + H_k^*H_k=1 \) and define the function

$$\begin{aligned} \varphi (A_1,\dots ,A_k)=\mathrm{Tr}~\exp _q\left( \sum _{i=1}^k H_i^*\log _q(A_i) H_i\right) \end{aligned}$$
(3)

in \( k \)-tuples of positive definite matrices. Then \( \varphi \) is positively homogeneous of degree one. It is concave for \( 1\le q\le 2 \) and convex for \( 2\le q\le 3. \)

Proof

For \( q>1 \) we obtain

$$\begin{aligned} \varphi (A_1,\dots , A_k)= & {} \displaystyle \mathrm{Tr}~\exp _q\left( \sum _{i=1}^k H_i^*\log _q(A_i) H_i\right) \\= & {} \displaystyle \mathrm{Tr}~\left( 1+(q-1)\sum _{i=1}^k H_i^*\log _q(A_i)H_i\right) ^{1/(q-1)}\\= & {} \displaystyle \mathrm{Tr}~\left( 1+(q-1)\sum _{i=1}^k H_i^*\frac{A_i^{q-1}-1}{q-1}H_i\right) ^{1/(q-1)}\\= & {} \displaystyle \mathrm{Tr}~\left( 1+\sum _{i=1}^k H_i^*(A_i^{q-1} -1)H_i \right) ^{1/(q-1)}\\= & {} \displaystyle \mathrm{Tr}~\left( H_1^* A_1^{q-1}H_1+\cdots +H_k^*A_k^{q-1}H_k\right) ^{1/(q-1)}. \end{aligned}$$

From this identity it follows that \( \varphi \) is positively homogeneous of degree one. The concavity for \( 1<q\le 2 \) and the convexity for \( 2\le q\le 3 \) now follows from (2). The statement for \( q=1 \) follows by letting \( q \) tend to one. \(\square \)

Corollary 3.2

Let \( L \) be a positive definite matrix, and let \( H_1,\dots ,H_k \) be matrices such that \( H_1^*H_1 +\cdots + H_k^*H_k\le 1. \) Then the function

$$\begin{aligned} \varphi (A_1,\dots ,A_k)=\mathrm{Tr}~\exp _q\left( L+H_1^*\log _q(A_1)H_1+\cdots +H_k^*\log _q(A_k)H_k\right) , \end{aligned}$$

defined in \( k \)-tuples of positive definite matrices, is concave for \( 1\le q\le 2 \) and convex for \( 2\le q\le 3. \)

Proof

We may without loss of generality assume \( H_1^*H_1 +\cdots + H_k^*H_k< 1 \) and put \( H_{k+1}=\left( 1-( H_1^*H_1 +\cdots + H_k^*H_k)\right) ^{1/2}. \) We then have

$$\begin{aligned} H_1^*H_1+\cdots +H_k^*H_k+H_{k+1}^*H_{k+1}=1 \end{aligned}$$

and may use the preceding theorem to conclude that the function

$$\begin{aligned} \begin{array}{l} (A_1,\dots ,A_{k+1})\rightarrow \mathrm{Tr}~\exp _q\left( H_1^*\log _q(A_1)H_1+\cdots +H_{k+1}^*\log _q(A_{k+1})H_{k+1}\right) \end{array} \end{aligned}$$

of \( k+1 \) variables is concave for \( 1\le q\le 2 \) and convex for \( 2\le q\le 3. \) Since \( H_{k+1} \) is invertible we may choose

$$\begin{aligned} A_{k+1}=\exp _q\left( H_{k+1}^{-1}LH_{k+1}^{-1}\right) \end{aligned}$$

which makes sense since \( H_{k+1}^{-1}LH_{k+1}^{-1} \) is positive definite. Concavity for \( 1\le q\le 2 \) and convexity for \( 2\le q\le 3 \) in the first \( k \) variables of the above function then yields the result. \(\square \)

Setting \( q=1 \) we recover in particular [5, Theorem 3].

Corollary 3.3

Let \( H_1,\dots ,H_k \) be matrices with \( H_1^*H_1+\cdots +H_k^*H_k\le 1, \) and let \( L \) be self-adjoint. The trace function

$$\begin{aligned} (A_1,\dots ,A_k)\rightarrow \mathrm{Tr}~\exp \left( L+H_1^*\log (A_1)H_1+\cdots +H_k^* \log (A_k)H_k\right) \end{aligned}$$

is concave in positive definite matrices.

Corollary 3.4

The trace function \( \varphi \) defined in (3) satisfies

$$\begin{aligned} \varphi (B_1,\dots ,B_k)\le \displaystyle \mathrm{Tr}~\exp _q \left( \sum _{i=1}^k H_i^*\log _q(A_i) H_i\right) ^{2-q} \sum _{j=1}^k H_{j}^*(d\!{}\log _q (A_j)B_j) H_j \end{aligned}$$

for \( 1\le q\le 2 \) and

$$\begin{aligned} \varphi (B_1,\dots ,B_k)\ge \displaystyle \mathrm{Tr}~\exp _q \left( \sum _{i=1}^k H_i^*\log _q(A_i) H_i\right) ^{2-q} \sum _{j=1}^k H_j^*(d\!{}\log _q(A_j)B_j) H_j \end{aligned}$$

for \( 2\le q\le 3, \) where \( A_1,\dots ,A_k \) and \( B_1,\dots ,B_k \) are positive definite matrices.

Proof

For \( 1\le q\le 2 \) we obtain

$$\begin{aligned} d\!{}\varphi (A_1,\dots ,A_k)(B_1,\dots ,B_k)\ge \varphi (B_1,\dots ,B_k) \end{aligned}$$

by Lemma  2.1. By the chain rule for Fréchet differentiable mappings between Banach spaces we therefore obtain

$$\begin{aligned} \varphi (B_1,\dots ,B_k)\le & {} \displaystyle \sum _{j=1}^k d_j \varphi (A_1,\dots ,A_k)B_j \\= & {} \displaystyle \sum _{j=1}^k \mathrm{Tr}~d\!{}\exp _q \left( \sum _{i=1}^k H_i^*\log _q(A_i) H_i\right) H_j^*(d\!{}\log _q(A_j)B_j) H_j\\= & {} \displaystyle \sum _{j=1}^k \mathrm{Tr}~\exp _q \left( \sum _{i=1}^k H_i^*\log _q(A_i) H_i\right) ^{2-q} H_j^*(d\!{}\log _q(A_j)B_j) H_j \end{aligned}$$

where we used the identity \( \mathrm{Tr}~d\!{}f(A)B=\mathrm{Tr}~f'(A)B \) valid for differentiable functions. This proves the first assertion. The result for \( 2\le q\le 3 \) follows similarly. \(\square \)

4 Proof of the Main Theorem

In order to prove Theorem 1.1 i we set \( k=2 \) in Corollary 3.4 and obtain

$$\begin{aligned} \varphi (B_1,B_2)\le \mathrm{Tr}~\exp _q(X)^{2-q}\left( H_1^* (d\!{}\log _q(A_1)B_1) H_1 + H_2^* (d\!{}\log _q(A_2)B_2) H_2\right) \end{aligned}$$

for \( 1\le q\le 2 \) and positive definite matrices \( A_1,A_2 \) and \( B_1,B_2 \) where

$$\begin{aligned} X=H_1^*\log _q(A_1) H_1 + H_2^* \log _q(A_2) H_2\,. \end{aligned}$$

If we set \( A_1=B_1 \) and \( A_2=1 \) the inequality reduces to

$$\begin{aligned} \varphi (B_1,B_2)\le \mathrm{Tr}~\exp _q(H_1^* \log _q(B_1) H_1)^{2-q}\left( H_1^* B_1^{q-1} H_1 +H_2^* B_2 H_2\right) . \end{aligned}$$

We now set \( H_1=\varepsilon ^{1/2} \) for \( 0<\varepsilon <1, \) and to fixed positive definite matrices \( L_1 \) and \( L_2 \) we choose \( B_1 \) and \( B_2 \) such that

$$\begin{aligned} \begin{array}{rl} L_1&{}=H_1^*\log _q(B_1) H_1=\varepsilon \log _q(B_1)\\ L_2&{}=H_2^*\log _q(B_2) H_2=(1-\varepsilon )\log _q(B_2). \end{array} \end{aligned}$$

It follows that

$$\begin{aligned} B_1=\exp _q\left( \varepsilon ^{-1}L_1\right) \qquad \text {and}\qquad B_2=\exp _q\left( (1-\varepsilon )^{-1}L_2\right) . \end{aligned}$$

Inserting in the inequality we obtain

$$\begin{aligned} \mathrm{Tr}~\exp _q(L_1+L_2)\le & {} \mathrm{Tr}~\exp _q(L_1)^{2-q}\left( \varepsilon \exp _q(\varepsilon ^{-1}L_1)^{q-1}+(1-\varepsilon )\exp _q((1-\varepsilon )^{-1}L_2)\right) \\= & {} \mathrm{Tr}~\exp _q(L_1)^{2-q}\left( L_1(q-1)+\varepsilon +(1-\varepsilon )\exp _q((1-\varepsilon )^{-1}L_2)\right) . \end{aligned}$$

This expression decouble \( L_1 \) and \( L_2 \) and reduces the minimisation problem over \( \varepsilon \) to the commutative case. We furthermore realise that minimum is obtained by letting \( \varepsilon \) tend to zero and that

$$\begin{aligned} \lim _{\varepsilon \rightarrow 0} (1-\varepsilon )\exp _q\left( (1-\varepsilon )^{-1}L_2\right) =\exp _q(L_2). \end{aligned}$$

We finally replace \( L_1 \) and \( L_2 \) with \( A \) and \( B. \) This proves the first statement in Theorem 1.1.

The proof of the second statement is virtually identical to the proof of the first. Since now \( 2\le q\le 3 \) the second inequality in Corollary 3.4 applies. Setting \( k=2 \) and applying the same substitutions as in the proof of the first statement we arrive at the inequality

$$\begin{aligned} \begin{array}{l} \mathrm{Tr}~\exp _q(L_1+L_2)\ge \mathrm{Tr}~\exp _q(L_1)^{2-q}\left( L_1(q-1)+\varepsilon +(1-\varepsilon )\exp _q((1-\varepsilon )^{-1}L_2)\right) . \end{array} \end{aligned}$$

Since \( 2\le q\le 3 \) the function

$$\begin{aligned} \varepsilon \rightarrow \varepsilon +(1-\varepsilon )\exp _q\left( (1-\varepsilon )^{-1}L_2\right) \end{aligned}$$

is now decreasing, and we thus maximise the right hand side in the above inequality by letting \( \varepsilon \) tend to zero. This proves the second statement in Theorem 1.1.